Contour integration when pole is outside the contour












0












$begingroup$


Contour integral with pole outside



Here they are using the pole OUTSIDE the contour? I thought this was illegal according to the residue theorem or we are not supposed to do contour integration with poles outside the contour itself.










share|cite|improve this question









$endgroup$












  • $begingroup$
    Which pole do you mean? They are not using the pole at zero, they are noticing that there are relevant poles because of the fact that the pole at zero isn't inside the contour.
    $endgroup$
    – Ian
    Dec 23 '14 at 19:51












  • $begingroup$
    So they're not using the residue theorem?
    $endgroup$
    – Amad27
    Dec 23 '14 at 20:01










  • $begingroup$
    Well, they do: the residue theorem says the integral over the contour is $0$.
    $endgroup$
    – Daniel Fischer
    Dec 23 '14 at 20:04










  • $begingroup$
    @DanielFischer, how? Where??
    $endgroup$
    – Amad27
    Dec 23 '14 at 20:07






  • 1




    $begingroup$
    There is no use of the Residue theorem here. It's Cauchy's integral Theorem that the integral is zero. This is because the integrand is analytic inside of the closed contour because there are no singularities there. Next, they parameterize the contour accordingly. The rest of the problem shouldn't be too bad.
    $endgroup$
    – DaveNine
    Dec 23 '14 at 21:45


















0












$begingroup$


Contour integral with pole outside



Here they are using the pole OUTSIDE the contour? I thought this was illegal according to the residue theorem or we are not supposed to do contour integration with poles outside the contour itself.










share|cite|improve this question









$endgroup$












  • $begingroup$
    Which pole do you mean? They are not using the pole at zero, they are noticing that there are relevant poles because of the fact that the pole at zero isn't inside the contour.
    $endgroup$
    – Ian
    Dec 23 '14 at 19:51












  • $begingroup$
    So they're not using the residue theorem?
    $endgroup$
    – Amad27
    Dec 23 '14 at 20:01










  • $begingroup$
    Well, they do: the residue theorem says the integral over the contour is $0$.
    $endgroup$
    – Daniel Fischer
    Dec 23 '14 at 20:04










  • $begingroup$
    @DanielFischer, how? Where??
    $endgroup$
    – Amad27
    Dec 23 '14 at 20:07






  • 1




    $begingroup$
    There is no use of the Residue theorem here. It's Cauchy's integral Theorem that the integral is zero. This is because the integrand is analytic inside of the closed contour because there are no singularities there. Next, they parameterize the contour accordingly. The rest of the problem shouldn't be too bad.
    $endgroup$
    – DaveNine
    Dec 23 '14 at 21:45
















0












0








0





$begingroup$


Contour integral with pole outside



Here they are using the pole OUTSIDE the contour? I thought this was illegal according to the residue theorem or we are not supposed to do contour integration with poles outside the contour itself.










share|cite|improve this question









$endgroup$




Contour integral with pole outside



Here they are using the pole OUTSIDE the contour? I thought this was illegal according to the residue theorem or we are not supposed to do contour integration with poles outside the contour itself.







calculus integration complex-analysis improper-integrals contour-integration






share|cite|improve this question













share|cite|improve this question











share|cite|improve this question




share|cite|improve this question










asked Dec 23 '14 at 19:49









Amad27Amad27

5,32821853




5,32821853












  • $begingroup$
    Which pole do you mean? They are not using the pole at zero, they are noticing that there are relevant poles because of the fact that the pole at zero isn't inside the contour.
    $endgroup$
    – Ian
    Dec 23 '14 at 19:51












  • $begingroup$
    So they're not using the residue theorem?
    $endgroup$
    – Amad27
    Dec 23 '14 at 20:01










  • $begingroup$
    Well, they do: the residue theorem says the integral over the contour is $0$.
    $endgroup$
    – Daniel Fischer
    Dec 23 '14 at 20:04










  • $begingroup$
    @DanielFischer, how? Where??
    $endgroup$
    – Amad27
    Dec 23 '14 at 20:07






  • 1




    $begingroup$
    There is no use of the Residue theorem here. It's Cauchy's integral Theorem that the integral is zero. This is because the integrand is analytic inside of the closed contour because there are no singularities there. Next, they parameterize the contour accordingly. The rest of the problem shouldn't be too bad.
    $endgroup$
    – DaveNine
    Dec 23 '14 at 21:45




















  • $begingroup$
    Which pole do you mean? They are not using the pole at zero, they are noticing that there are relevant poles because of the fact that the pole at zero isn't inside the contour.
    $endgroup$
    – Ian
    Dec 23 '14 at 19:51












  • $begingroup$
    So they're not using the residue theorem?
    $endgroup$
    – Amad27
    Dec 23 '14 at 20:01










  • $begingroup$
    Well, they do: the residue theorem says the integral over the contour is $0$.
    $endgroup$
    – Daniel Fischer
    Dec 23 '14 at 20:04










  • $begingroup$
    @DanielFischer, how? Where??
    $endgroup$
    – Amad27
    Dec 23 '14 at 20:07






  • 1




    $begingroup$
    There is no use of the Residue theorem here. It's Cauchy's integral Theorem that the integral is zero. This is because the integrand is analytic inside of the closed contour because there are no singularities there. Next, they parameterize the contour accordingly. The rest of the problem shouldn't be too bad.
    $endgroup$
    – DaveNine
    Dec 23 '14 at 21:45


















$begingroup$
Which pole do you mean? They are not using the pole at zero, they are noticing that there are relevant poles because of the fact that the pole at zero isn't inside the contour.
$endgroup$
– Ian
Dec 23 '14 at 19:51






$begingroup$
Which pole do you mean? They are not using the pole at zero, they are noticing that there are relevant poles because of the fact that the pole at zero isn't inside the contour.
$endgroup$
– Ian
Dec 23 '14 at 19:51














$begingroup$
So they're not using the residue theorem?
$endgroup$
– Amad27
Dec 23 '14 at 20:01




$begingroup$
So they're not using the residue theorem?
$endgroup$
– Amad27
Dec 23 '14 at 20:01












$begingroup$
Well, they do: the residue theorem says the integral over the contour is $0$.
$endgroup$
– Daniel Fischer
Dec 23 '14 at 20:04




$begingroup$
Well, they do: the residue theorem says the integral over the contour is $0$.
$endgroup$
– Daniel Fischer
Dec 23 '14 at 20:04












$begingroup$
@DanielFischer, how? Where??
$endgroup$
– Amad27
Dec 23 '14 at 20:07




$begingroup$
@DanielFischer, how? Where??
$endgroup$
– Amad27
Dec 23 '14 at 20:07




1




1




$begingroup$
There is no use of the Residue theorem here. It's Cauchy's integral Theorem that the integral is zero. This is because the integrand is analytic inside of the closed contour because there are no singularities there. Next, they parameterize the contour accordingly. The rest of the problem shouldn't be too bad.
$endgroup$
– DaveNine
Dec 23 '14 at 21:45






$begingroup$
There is no use of the Residue theorem here. It's Cauchy's integral Theorem that the integral is zero. This is because the integrand is analytic inside of the closed contour because there are no singularities there. Next, they parameterize the contour accordingly. The rest of the problem shouldn't be too bad.
$endgroup$
– DaveNine
Dec 23 '14 at 21:45












1 Answer
1






active

oldest

votes


















0












$begingroup$

In this case you can use the Sokhotski–Plemelj theorem and work with the principal value directly. This amounts to putting the pole right on the contour and counting it as being "half inside" making the contribution $pi i$ times the residue. In general, a contour integral can be written as minus $2pi i$ times the sum of all the residues of the poles outside the contour, but you then also need to include the residue of the pole at infinity (if there is a pole there). To see this consider an arbitrary contour integral of a meromorphic function $f(z)$:



$$I = oint_C f(z) dz$$



where $C$ is a counterclockwise contour. Suppose that a point $p$ is inside the contour. We can then define a new variable $w$ by putting:



$$w = frac{1}{z - p}$$



Performing this change of variables to the integral yields:



$$I = oint_{C_w} frac{1}{w^2}fleft(frac{1}{w}+pright) dw$$



Note that the transform maps the interior of the contour to the exterior and vice versa, the winding number of the contour reverses, but we absorbed a minus sign from the transform of $d$z to $dw$ to keep the new contour counterclockwise.



Suppose then that $f(z)$ has a pole of order m at $z = q$. This will then correspond to a pole at $w = q_w = frac{1}{q-p}$ of $fleft(frac{1}{w}+pright)$. The Laurent expansion of $f(z)$ transforms to the Laurent expansion for $frac{1}{w^2}fleft(frac{1}{w}+pright)$ as follows. A term $(z-q)^{-n}$ transforms to the term $(-1)^n q_w^n w^{n-2}(w-q_w)^{-n}$. One then has to expand the factor $w^{n-2}$ around $q_w$ to obtain the transformed Laurent expansion. But note that for $n>1$ one obtains a polynomial of degree $n-2$ multiplying the term $(w-q_w)^{-n}$, this then yields contribution up to the term $(w-q_w)^{-2}$ but not to higher powers, so the residue is not affected by such terms. The residue is thus obtained from only the term $(-1)^n q_w^n w^{n-2}(w-q_w)^{-n}$ for $n = 1$ expanded around $q_w$, this yields the term $-frac{1}{w-q_w}$. This means that the residue of the new integrand changes sign relative to original residue of $f(z)$.



We know that the integral is the sum of $2pi i$ times the sum of the residues of the poles inside the contour, if we apply this to the transformed integral we see that this is the same as minus $2pi i$ times the sum of the residues of the poles outside the original contour of the original integral. But this is only true if there is no pole at $w = 0$. If there is a pole there then we define the residue at that point as minus the residue of $f(z)$ at infinity. This then makes the statement that the integral is minus the sum of the residues outside the contour true in general.






share|cite|improve this answer









$endgroup$













    Your Answer





    StackExchange.ifUsing("editor", function () {
    return StackExchange.using("mathjaxEditing", function () {
    StackExchange.MarkdownEditor.creationCallbacks.add(function (editor, postfix) {
    StackExchange.mathjaxEditing.prepareWmdForMathJax(editor, postfix, [["$", "$"], ["\\(","\\)"]]);
    });
    });
    }, "mathjax-editing");

    StackExchange.ready(function() {
    var channelOptions = {
    tags: "".split(" "),
    id: "69"
    };
    initTagRenderer("".split(" "), "".split(" "), channelOptions);

    StackExchange.using("externalEditor", function() {
    // Have to fire editor after snippets, if snippets enabled
    if (StackExchange.settings.snippets.snippetsEnabled) {
    StackExchange.using("snippets", function() {
    createEditor();
    });
    }
    else {
    createEditor();
    }
    });

    function createEditor() {
    StackExchange.prepareEditor({
    heartbeatType: 'answer',
    autoActivateHeartbeat: false,
    convertImagesToLinks: true,
    noModals: true,
    showLowRepImageUploadWarning: true,
    reputationToPostImages: 10,
    bindNavPrevention: true,
    postfix: "",
    imageUploader: {
    brandingHtml: "Powered by u003ca class="icon-imgur-white" href="https://imgur.com/"u003eu003c/au003e",
    contentPolicyHtml: "User contributions licensed under u003ca href="https://creativecommons.org/licenses/by-sa/3.0/"u003ecc by-sa 3.0 with attribution requiredu003c/au003e u003ca href="https://stackoverflow.com/legal/content-policy"u003e(content policy)u003c/au003e",
    allowUrls: true
    },
    noCode: true, onDemand: true,
    discardSelector: ".discard-answer"
    ,immediatelyShowMarkdownHelp:true
    });


    }
    });














    draft saved

    draft discarded


















    StackExchange.ready(
    function () {
    StackExchange.openid.initPostLogin('.new-post-login', 'https%3a%2f%2fmath.stackexchange.com%2fquestions%2f1079114%2fcontour-integration-when-pole-is-outside-the-contour%23new-answer', 'question_page');
    }
    );

    Post as a guest















    Required, but never shown

























    1 Answer
    1






    active

    oldest

    votes








    1 Answer
    1






    active

    oldest

    votes









    active

    oldest

    votes






    active

    oldest

    votes









    0












    $begingroup$

    In this case you can use the Sokhotski–Plemelj theorem and work with the principal value directly. This amounts to putting the pole right on the contour and counting it as being "half inside" making the contribution $pi i$ times the residue. In general, a contour integral can be written as minus $2pi i$ times the sum of all the residues of the poles outside the contour, but you then also need to include the residue of the pole at infinity (if there is a pole there). To see this consider an arbitrary contour integral of a meromorphic function $f(z)$:



    $$I = oint_C f(z) dz$$



    where $C$ is a counterclockwise contour. Suppose that a point $p$ is inside the contour. We can then define a new variable $w$ by putting:



    $$w = frac{1}{z - p}$$



    Performing this change of variables to the integral yields:



    $$I = oint_{C_w} frac{1}{w^2}fleft(frac{1}{w}+pright) dw$$



    Note that the transform maps the interior of the contour to the exterior and vice versa, the winding number of the contour reverses, but we absorbed a minus sign from the transform of $d$z to $dw$ to keep the new contour counterclockwise.



    Suppose then that $f(z)$ has a pole of order m at $z = q$. This will then correspond to a pole at $w = q_w = frac{1}{q-p}$ of $fleft(frac{1}{w}+pright)$. The Laurent expansion of $f(z)$ transforms to the Laurent expansion for $frac{1}{w^2}fleft(frac{1}{w}+pright)$ as follows. A term $(z-q)^{-n}$ transforms to the term $(-1)^n q_w^n w^{n-2}(w-q_w)^{-n}$. One then has to expand the factor $w^{n-2}$ around $q_w$ to obtain the transformed Laurent expansion. But note that for $n>1$ one obtains a polynomial of degree $n-2$ multiplying the term $(w-q_w)^{-n}$, this then yields contribution up to the term $(w-q_w)^{-2}$ but not to higher powers, so the residue is not affected by such terms. The residue is thus obtained from only the term $(-1)^n q_w^n w^{n-2}(w-q_w)^{-n}$ for $n = 1$ expanded around $q_w$, this yields the term $-frac{1}{w-q_w}$. This means that the residue of the new integrand changes sign relative to original residue of $f(z)$.



    We know that the integral is the sum of $2pi i$ times the sum of the residues of the poles inside the contour, if we apply this to the transformed integral we see that this is the same as minus $2pi i$ times the sum of the residues of the poles outside the original contour of the original integral. But this is only true if there is no pole at $w = 0$. If there is a pole there then we define the residue at that point as minus the residue of $f(z)$ at infinity. This then makes the statement that the integral is minus the sum of the residues outside the contour true in general.






    share|cite|improve this answer









    $endgroup$


















      0












      $begingroup$

      In this case you can use the Sokhotski–Plemelj theorem and work with the principal value directly. This amounts to putting the pole right on the contour and counting it as being "half inside" making the contribution $pi i$ times the residue. In general, a contour integral can be written as minus $2pi i$ times the sum of all the residues of the poles outside the contour, but you then also need to include the residue of the pole at infinity (if there is a pole there). To see this consider an arbitrary contour integral of a meromorphic function $f(z)$:



      $$I = oint_C f(z) dz$$



      where $C$ is a counterclockwise contour. Suppose that a point $p$ is inside the contour. We can then define a new variable $w$ by putting:



      $$w = frac{1}{z - p}$$



      Performing this change of variables to the integral yields:



      $$I = oint_{C_w} frac{1}{w^2}fleft(frac{1}{w}+pright) dw$$



      Note that the transform maps the interior of the contour to the exterior and vice versa, the winding number of the contour reverses, but we absorbed a minus sign from the transform of $d$z to $dw$ to keep the new contour counterclockwise.



      Suppose then that $f(z)$ has a pole of order m at $z = q$. This will then correspond to a pole at $w = q_w = frac{1}{q-p}$ of $fleft(frac{1}{w}+pright)$. The Laurent expansion of $f(z)$ transforms to the Laurent expansion for $frac{1}{w^2}fleft(frac{1}{w}+pright)$ as follows. A term $(z-q)^{-n}$ transforms to the term $(-1)^n q_w^n w^{n-2}(w-q_w)^{-n}$. One then has to expand the factor $w^{n-2}$ around $q_w$ to obtain the transformed Laurent expansion. But note that for $n>1$ one obtains a polynomial of degree $n-2$ multiplying the term $(w-q_w)^{-n}$, this then yields contribution up to the term $(w-q_w)^{-2}$ but not to higher powers, so the residue is not affected by such terms. The residue is thus obtained from only the term $(-1)^n q_w^n w^{n-2}(w-q_w)^{-n}$ for $n = 1$ expanded around $q_w$, this yields the term $-frac{1}{w-q_w}$. This means that the residue of the new integrand changes sign relative to original residue of $f(z)$.



      We know that the integral is the sum of $2pi i$ times the sum of the residues of the poles inside the contour, if we apply this to the transformed integral we see that this is the same as minus $2pi i$ times the sum of the residues of the poles outside the original contour of the original integral. But this is only true if there is no pole at $w = 0$. If there is a pole there then we define the residue at that point as minus the residue of $f(z)$ at infinity. This then makes the statement that the integral is minus the sum of the residues outside the contour true in general.






      share|cite|improve this answer









      $endgroup$
















        0












        0








        0





        $begingroup$

        In this case you can use the Sokhotski–Plemelj theorem and work with the principal value directly. This amounts to putting the pole right on the contour and counting it as being "half inside" making the contribution $pi i$ times the residue. In general, a contour integral can be written as minus $2pi i$ times the sum of all the residues of the poles outside the contour, but you then also need to include the residue of the pole at infinity (if there is a pole there). To see this consider an arbitrary contour integral of a meromorphic function $f(z)$:



        $$I = oint_C f(z) dz$$



        where $C$ is a counterclockwise contour. Suppose that a point $p$ is inside the contour. We can then define a new variable $w$ by putting:



        $$w = frac{1}{z - p}$$



        Performing this change of variables to the integral yields:



        $$I = oint_{C_w} frac{1}{w^2}fleft(frac{1}{w}+pright) dw$$



        Note that the transform maps the interior of the contour to the exterior and vice versa, the winding number of the contour reverses, but we absorbed a minus sign from the transform of $d$z to $dw$ to keep the new contour counterclockwise.



        Suppose then that $f(z)$ has a pole of order m at $z = q$. This will then correspond to a pole at $w = q_w = frac{1}{q-p}$ of $fleft(frac{1}{w}+pright)$. The Laurent expansion of $f(z)$ transforms to the Laurent expansion for $frac{1}{w^2}fleft(frac{1}{w}+pright)$ as follows. A term $(z-q)^{-n}$ transforms to the term $(-1)^n q_w^n w^{n-2}(w-q_w)^{-n}$. One then has to expand the factor $w^{n-2}$ around $q_w$ to obtain the transformed Laurent expansion. But note that for $n>1$ one obtains a polynomial of degree $n-2$ multiplying the term $(w-q_w)^{-n}$, this then yields contribution up to the term $(w-q_w)^{-2}$ but not to higher powers, so the residue is not affected by such terms. The residue is thus obtained from only the term $(-1)^n q_w^n w^{n-2}(w-q_w)^{-n}$ for $n = 1$ expanded around $q_w$, this yields the term $-frac{1}{w-q_w}$. This means that the residue of the new integrand changes sign relative to original residue of $f(z)$.



        We know that the integral is the sum of $2pi i$ times the sum of the residues of the poles inside the contour, if we apply this to the transformed integral we see that this is the same as minus $2pi i$ times the sum of the residues of the poles outside the original contour of the original integral. But this is only true if there is no pole at $w = 0$. If there is a pole there then we define the residue at that point as minus the residue of $f(z)$ at infinity. This then makes the statement that the integral is minus the sum of the residues outside the contour true in general.






        share|cite|improve this answer









        $endgroup$



        In this case you can use the Sokhotski–Plemelj theorem and work with the principal value directly. This amounts to putting the pole right on the contour and counting it as being "half inside" making the contribution $pi i$ times the residue. In general, a contour integral can be written as minus $2pi i$ times the sum of all the residues of the poles outside the contour, but you then also need to include the residue of the pole at infinity (if there is a pole there). To see this consider an arbitrary contour integral of a meromorphic function $f(z)$:



        $$I = oint_C f(z) dz$$



        where $C$ is a counterclockwise contour. Suppose that a point $p$ is inside the contour. We can then define a new variable $w$ by putting:



        $$w = frac{1}{z - p}$$



        Performing this change of variables to the integral yields:



        $$I = oint_{C_w} frac{1}{w^2}fleft(frac{1}{w}+pright) dw$$



        Note that the transform maps the interior of the contour to the exterior and vice versa, the winding number of the contour reverses, but we absorbed a minus sign from the transform of $d$z to $dw$ to keep the new contour counterclockwise.



        Suppose then that $f(z)$ has a pole of order m at $z = q$. This will then correspond to a pole at $w = q_w = frac{1}{q-p}$ of $fleft(frac{1}{w}+pright)$. The Laurent expansion of $f(z)$ transforms to the Laurent expansion for $frac{1}{w^2}fleft(frac{1}{w}+pright)$ as follows. A term $(z-q)^{-n}$ transforms to the term $(-1)^n q_w^n w^{n-2}(w-q_w)^{-n}$. One then has to expand the factor $w^{n-2}$ around $q_w$ to obtain the transformed Laurent expansion. But note that for $n>1$ one obtains a polynomial of degree $n-2$ multiplying the term $(w-q_w)^{-n}$, this then yields contribution up to the term $(w-q_w)^{-2}$ but not to higher powers, so the residue is not affected by such terms. The residue is thus obtained from only the term $(-1)^n q_w^n w^{n-2}(w-q_w)^{-n}$ for $n = 1$ expanded around $q_w$, this yields the term $-frac{1}{w-q_w}$. This means that the residue of the new integrand changes sign relative to original residue of $f(z)$.



        We know that the integral is the sum of $2pi i$ times the sum of the residues of the poles inside the contour, if we apply this to the transformed integral we see that this is the same as minus $2pi i$ times the sum of the residues of the poles outside the original contour of the original integral. But this is only true if there is no pole at $w = 0$. If there is a pole there then we define the residue at that point as minus the residue of $f(z)$ at infinity. This then makes the statement that the integral is minus the sum of the residues outside the contour true in general.







        share|cite|improve this answer












        share|cite|improve this answer



        share|cite|improve this answer










        answered Jun 15 '16 at 20:51









        Count IblisCount Iblis

        8,24121533




        8,24121533






























            draft saved

            draft discarded




















































            Thanks for contributing an answer to Mathematics Stack Exchange!


            • Please be sure to answer the question. Provide details and share your research!

            But avoid



            • Asking for help, clarification, or responding to other answers.

            • Making statements based on opinion; back them up with references or personal experience.


            Use MathJax to format equations. MathJax reference.


            To learn more, see our tips on writing great answers.




            draft saved


            draft discarded














            StackExchange.ready(
            function () {
            StackExchange.openid.initPostLogin('.new-post-login', 'https%3a%2f%2fmath.stackexchange.com%2fquestions%2f1079114%2fcontour-integration-when-pole-is-outside-the-contour%23new-answer', 'question_page');
            }
            );

            Post as a guest















            Required, but never shown





















































            Required, but never shown














            Required, but never shown












            Required, but never shown







            Required, but never shown

































            Required, but never shown














            Required, but never shown












            Required, but never shown







            Required, but never shown







            Popular posts from this blog

            Mario Kart Wii

            What does “Dominus providebit” mean?

            Antonio Litta Visconti Arese